Van der Pol oscillator + Hopf bifurcation

Click For Summary
The discussion focuses on analyzing the biased van der Pol oscillator and identifying the conditions for Hopf bifurcations in the (u,a) parameter space. Participants share their approaches, with one successfully converting the system into a 2D format and deriving the Jacobian to find eigenvalues. The definition of a Hopf bifurcation is clarified, emphasizing the transition of eigenvalues from negative to positive real components. There are corrections made regarding the original equation's formulation, ensuring all participants are aligned on the problem. The conversation highlights the importance of fixed points and eigenvalue analysis in determining bifurcation conditions.
nicksauce
Science Advisor
Homework Helper
Messages
1,270
Reaction score
7

Homework Statement


Consider the biased van der Pol oscillator: \frac{d^2x}{dt^2}=u (x^2-1)\frac{dx}{dt} + x = a. Find the curves in (u,a) space at which Hopf bifurcations occur. (Strogatz 8.21)


Homework Equations





The Attempt at a Solution


Not even sure where to start with this question, other than turning it into a 2-d system. Any advice on how to approach this?
 
Physics news on Phys.org
I'm attempting a very similar question. I've managed to turn it into a 2D system but haven't got much further.

dx/dt = y

dy/dt = u(x^2 -1)y + x

A Hopf bifurcation is one where, as you vary u, the eigenvalues of the Jacobian change from having a negative real component to having a positive real component.

I'll let you know if I make more progress.
 
I've just looked in Strogatz at question 8.21 and I think you've copied out the question incorrectly:
you wrote an equals sign between d^2x/dt^2 and u(x^2 -1)
but Strogratz wrote a plus.

Luckily, this means we're working on exactly the same question.
The equations for the 2D system are not as written above; insteat they are:

dx/dt = y

dy/dt = a - x - u(x^2 -1)y
 
I think I've answered the question. My method was as follows:

I wrote down the Jacobian and got an expression for its eigenvalues. I then found the fixed points. I found what the eigenvalues are at the fixed points. It's then just a matter of inspecting your solution, looking at the definition of a Hopf bifurcation and seeing what values u and a need to take.

Best of luck,
quantum boy
 
i took did the jacobian and got eigenvalues of 0 and u(a^2)-u...cause i found a fixed point of (a,0) from nullclines...now what do I do...
 
Question: A clock's minute hand has length 4 and its hour hand has length 3. What is the distance between the tips at the moment when it is increasing most rapidly?(Putnam Exam Question) Answer: Making assumption that both the hands moves at constant angular velocities, the answer is ## \sqrt{7} .## But don't you think this assumption is somewhat doubtful and wrong?

Similar threads

  • · Replies 4 ·
Replies
4
Views
2K
  • · Replies 105 ·
4
Replies
105
Views
6K
  • · Replies 1 ·
Replies
1
Views
2K
  • · Replies 1 ·
Replies
1
Views
3K
  • · Replies 4 ·
Replies
4
Views
2K
  • · Replies 22 ·
Replies
22
Views
3K
Replies
6
Views
2K
  • · Replies 1 ·
Replies
1
Views
1K
  • · Replies 23 ·
Replies
23
Views
3K
Replies
1
Views
2K